1answer.
Ask question
Login Signup
Ask question
All categories
  • English
  • Mathematics
  • Social Studies
  • Business
  • History
  • Health
  • Geography
  • Biology
  • Physics
  • Chemistry
  • Computers and Technology
  • Arts
  • World Languages
  • Spanish
  • French
  • German
  • Advanced Placement (AP)
  • SAT
  • Medicine
  • Law
  • Engineering
xxTIMURxx [149]
3 years ago
15

The coordinate of M is 2.5 and MN = 4 what are the possible coordinates for N

Mathematics
1 answer:
stira [4]3 years ago
3 0
The possible coordinates for N is 0
You might be interested in
At summer camp, 150 kids had the opportunity to choose their activities. 1/3 of those kids selected archery for their morning ac
KIM [24]
They choose 25 options on five kis
7 0
3 years ago
The LCM of three numbers is 7920 and their GCD is 12. Two of the numbers are 48 and 264. Using factor notation find the third nu
Lelechka [254]

Answer:

The third number is 180.

Step-by-step explanation:

Let the third number be 9x where x is an integer.

48 = 2*2*2*2*3

264 = 2*2*2*3*11

9x = 3*3*x

2*2*2*2*3*3*11*x = 7920y where y is an integer

1584x = 7920y

so x = 7920y / 1584= 5y.

Now the GCD is 12  so x must have  4 as one of its factors.

Also x is a multiple of 5 so it could be 20 then y  would be 4.

If x = 20 then the third number is 9*20 = 180.

This checks out OK.

6 0
3 years ago
Help 1 and 4 fast please
AlekseyPX

Answer:

1)

\leq : Less than or equal to\\\geq :Greater than or equal to\\:greater than

Step-by-step explanation:

N/A

also what is your inequality problem

3 0
3 years ago
Which is 5logx - 6log(x-8) written as a single logarithm?
ira [324]

Answer:

option a

log\frac{x^{5} }{(x-8)^{6} }

Step-by-step explanation:

Given in the question an expression

5logx - 6log(x-6)

To solve this question we will apply two of the logarithm property

1) power rule

alogx = logx^{n}

logx^{5} - log(x-8)^{6}

2) substraction rule

The log of a quotient is the difference of the logs

loga (x/y) = loga x - loga y

logx^{5}- log(x-8)^{6}

log\frac{x^{5} }{(x-8)^{6} }

6 0
3 years ago
Read 2 more answers
5,049 +48=105
Alborosie
<h2>Solution (a) :-</h2>

Let the cost of the less expensive comic book be x .

Then :-

1284 - 11x = x

Using this equation , we can find the cost of the less expensive comic book .

Price of the less expensive book :-

1284 - 11x = x

11x + x = 1284

12x = 1284

x =  \frac{1284}{12}

x = 107

Cost of the less expensive book is 107 .

Therefore , the epice of the less expensive book = $ 107 .

<h2>Solution (b) :-</h2>

Cost of the more expensive book = 11x

Which means :-

1284 - x = 11x

11x + x = 1284

12x = 1284

x = 107

11x = 11 \times 107

= 1177

Therefore , the price of the more expensive book = $ 1177 .

4 0
3 years ago
Other questions:
  • is this a relation or function ? what is the domain and range ? what is the input and output ? is it discrete or continuous ? ​
    12·1 answer
  • Plzzzzzz helppppp asappppp <br> ***will mark brainliest *****
    5·1 answer
  • Please help me with this
    7·1 answer
  • Can I get help on numbers 4,6,24, and 34 please
    6·1 answer
  • Kate bought a T-shirt for $12.95 and a pair of shorts for $10.95. She paid 6.25% in tax. What is her total amount that she paid?
    7·1 answer
  • 1
    7·1 answer
  • CAN ANYONE PLEASE HELP ME WITH MY HOMEWORK ILL DO ANYTHING JUST PLEASE HELP ME
    9·2 answers
  • =−2x^2 +8 +3<br> write in vertex form
    15·1 answer
  • Current Attempt in Progress
    6·1 answer
  • PLS PLS HELP ME THIS IS DUE TONIGHT
    13·2 answers
Add answer
Login
Not registered? Fast signup
Signup
Login Signup
Ask question!